Howdy, Stranger!

It looks like you're new here. If you want to get involved, click one of these buttons!

PT30.S4.Q20-- consumer advocate: the introduction of a new drug

DarklordDarklord Alum Member
edited March 2020 in Logical Reasoning 586 karma

Hi,

Based on what I have read from answer explanations so far, this logical reasoning question seems to break the "cardinal rule" of there only being one truly right answer because both A and B here seem to strengthen the stimulus-- A just happens to strengthen it more, and thus is the right answer. Does anyone have a better explanation for why B is wrong?

Thanks!

help

Admin note: https://7sage.com/lsat_explanations/lsat-30-section-4-question-20/

Comments

  • Hopeful9812Hopeful9812 Member
    872 karma

    I agree with you that A seems to strengthen the argument more. Here was my thought process for this question.
     
    Author's argument start at "however," the information before the however is context. The premise = The social impact of the newly marketed antihistamine is far from clear. The conclusion = There should be a general reduction in the pace of brining to the marketplace new drugs that are now being tested.
     
    Okay so the first thing that came to my mind was why should there be a general reduction in the pace of new drugs that are now being tested? In other words, how does the premise connect to the conclusion? So the premise is saying that we don't fully know the impact of antihistamines. But what if compared to new drugs, we know the impact of antihistamines better than that of new drugs. If that were true, then the conclusion makes sense that there should be a general reduction in the pace of new drugs. For this reason, A makes more sense and appears to be a more clear and direct answer choice.
     
    B was a strong contender but it is weaker compared to answer choice A. B says that the social impact of some of the new drugs being tested is poorly understood. If the impact of some new drugs are poorly understood, the possibility still remains that there may be other new drugs which are not poorly understood and if that is the case, then that doesn't really strengthen the conclusion like answer choice A does. I  think the other reason for B being weaker than A is that B doesn't directly mention antihistamines like how A does. In other words, B doesn't bridge that gap between antihistamines in the premise of the prompt and new drugs being marketed in the conclusion of the prompt.
     
    Not sure if any of this was helpful but this was just my thought process for this question!

  • DarklordDarklord Alum Member
    586 karma

    This was helpful-- thanks!

Sign In or Register to comment.